Đến nội dung

Nguyenhuyen_AG

Nguyenhuyen_AG

Đăng ký: 09-09-2010
Offline Đăng nhập: 10-01-2019 - 16:22
****-

#366098 CMR:$ \frac{1}{a^2-a+1}+\frac{1}...

Gửi bởi Nguyenhuyen_AG trong 30-10-2012 - 23:34

Điều bạn nói mình biết chứ :D nhưng trước đó thì Vũng Tàu đã lấy nó làm đề thi rồi :D Co về bất đẳng thức của Đào Hải Long, ta có thể giải đơn giản như sau. Giả sử $a>b>c,$ khi đó theo bất đẳng thức AM-GM, ta có $$\frac{1}{(a-b)^2}+\frac{1}{(b-c)^2}\ge \frac{2}{(a-b)(b-c)}\ge \frac{8}{(c-a)^2}. \quad (1)$$ Và theo bất đẳng thức Cauchy-Schwarz thì $$a^2+b^2+c^2 \ge a^2+(-c)^2 \ge \frac{1}{2}(a-c)^2. \quad (2)$$ Từ $(1)$ và $(2)$ ta được $$(a^2+b^2+c^2)\left [ \frac{1}{(a-b)^2}+\frac{1}{(b-c)^2}+\frac{1}{(c-a)^2} \right ]\ge \frac{1}{2}\cdot (a-c)^2\left [ \frac{8}{(a-c)^2}+\frac{1}{(a-c)^2} \right ]=\frac{9}{2}.$$
Ngoài ra ta còn có 5 cách khác nữa để chứng minh cho bài toán thú vị này.


#365994 $$(a^2+b^2+c^2)\left[\sum \frac{1}{(b...

Gửi bởi Nguyenhuyen_AG trong 30-10-2012 - 19:42

Một bài viết nhỉ của mình, về hai bài toán 1 và 3 trong chủ đề này. http://mathifc.wordp...-bat-dang-thuc/


#365989 $\frac{a^{2}}{3a^{2}+(b+c)^...

Gửi bởi Nguyenhuyen_AG trong 30-10-2012 - 19:25

Ta sẽ CM : $\frac{a^2}{3a^2+(b+c)^2}\leq \frac{a}{2(a+b+c)}$

Nếu $a=0$ thì bất đẳng thức hiển nhiên đúng, xét $a>0.$ Theo bất đẳng thức AM-GM, ta có $$\frac{a^2}{3a^2+(b+c)^2}=\frac{a^2}{2a^2+\left [a^2+(b+c)^2 \right ]}\leq \frac{a^2}{2a^2+2a(b+c)}=\frac{a}{2(a+b+c)}.$$


#365983 Cho $xy+yz+zx=1$. Tìm Min : $x^{2}+2y^{2}+...

Gửi bởi Nguyenhuyen_AG trong 30-10-2012 - 19:20

Cho $xy+yz+zx=1$. Tìm Min : $x^{2}+2y^{2}+5z^{2}$


Sử dụng bất đẳng thức Cauchy-Schwarz, ta có $$(2x^{2}+3y^{2}+6z^{2})\left ( \frac{1}{2}+\frac{1}{3}+\frac{1}{6} \right )\ge(x+y+z)^2.$$ Tức là $$2x^{2}+3y^{2}+6z^{2}\ge(x+y+z)^2,$$ hoặc $$x^{2}+2y^{2}+5z^{2} \ge 2(xy+yz+zx).$$ Đến đây, ta chỉ cần thay điều kiện của bài toán vào là được.
P/s. Sao bộ gõ chữ của diễn đàn nó bị nhảy tùm lum hết nhỉ :D Khó gõ quá.


#365982 $P = \left( {xy + yz + zx} \right).....$

Gửi bởi Nguyenhuyen_AG trong 30-10-2012 - 19:11

Ngoài ra, ta còn có một kết quả rất đẹp mắt sau đây, đặt $$F_k(a,b,c)=(ab+bc+ca)\left [ \frac{1}{a^2-kab+b^2}+\frac{1}{b^2-kbc+c^2}+\frac{1}{c^2-kca+a^2} \right ].$$ Khi đó, ta luôn có bất đẳng thức sau đây $$F_k(a,b,c)\ge \left\{\begin{matrix}\dfrac{5-2k}{2-k},\;\;\;(0\le k \le1)\\ 2+k,\;\;\;\;\;(1\le k\le2)\end{matrix}\right.$$


#365978 CMR:$ \frac{1}{a^2-a+1}+\frac{1}...

Gửi bởi Nguyenhuyen_AG trong 30-10-2012 - 18:54

Cho $a,b,c>0$. Tìm GTNN của$ P=(a^2+b^2+c^2).(\frac{1}{(a-b)^2}+\frac{1}{(b-c)^2}+\frac{1}{(c-a)^2}).$

Bạn cho cái điều kiện $a,b,c$ dương thì hơi bị khó ở :D, vì thực tế, nếu như $a,b,c$ là ba số thực khác nhau thì ta có $$(a^2+b^2+c^2)\left [ \frac{1}{(a-b)^2}+\frac{1}{(b-c)^2}+\frac{1}{(c-a)^2} \right ]\ge \frac{9}{2}.$$ Là một kết quả quen thuộc của Đào Hải Long, còn nếu như $a,b,c$ không âm đôi một khác nhau thì $$(a^2+b^2+c^2)\left [ \frac{1}{(a-b)^2}+\frac{1}{(b-c)^2}+\frac{1}{(c-a)^2} \right ]\ge \frac{11+5\sqrt{5}}{2}.$$


#365973 $\frac{a\left ( b+c \right )}{b^{2}+bc+c^{2}}+\frac{...

Gửi bởi Nguyenhuyen_AG trong 30-10-2012 - 18:35

Chú ý rằng
$$\frac{a(b+c)}{b^2+bc+c^2}=\frac{ab}{b^2+bc+c^2}+\frac{ca}{b^2+bc+c^2},$$
nên theo bất đẳng thức Cauchy-Schwarz, ta có
$$ \begin{aligned}\sum \frac{a(b+c)}{b^2+bc+c^2}& = \sum \frac{(a^2b)^2}{a^3b(b^2+bc+c^2)}+\sum \frac{(c^2a)^2}{c^3a(b^2+bc+c^2)} \\& \ge \frac{(a^2b+b^2c+c^2a)^2}{\displaystyle \sum a^3b(b^2+bc+c^2)}+\frac{(ca^2+ab^2+bc^2)^2}{\displaystyle \sum c^3a(b^2+bc+c^2)}\\&=\frac{(a^2b+b^2c+c^2a)^2+(ca^2+ab^2+bc^2)^2}{a^3b^3+b^3c^3+c^3a^3+abc[ab(a+b)+bc(b+c)+ca(c+a)]}.\end{aligned}$$ Kết hợp với bất đẳng thức Schur bậc 3 $$a^3+b^3+c^3+3abc\ge ab(a+b)+bc(b+c)+ca(c+a),$$
ta được
$$\begin{aligned}\sum \frac{a(b+c)}{b^2+bc+c^2} &\ge \frac{(a^2b+b^2c+c^2a)^2+(ca^2+ab^2+bc^2)^2}{a^3b^3+b^3c^3+c^3a^3+abc(a^3+b^3+c^3+3abc)}\\&=\frac{(a^2b+b^2c+c^2a)^2+(ca^2+ab^2+bc^2)^2}{(a^2b+b^2c+c^2a)(ca^2+ab^2+bc^2)}\\&=2+\frac{(a-b)^2(b-c)^2(c-a)^2}{(a^2b+b^2c+c^2a)(ab^2+bc^2+ca^2)}.\end{aligned}$$
Bài toán này được bạn Lê Việt Hải làm mạnh từ một bất đẳng thức của Darij Grinberg, ngoài ra ta cũng có một kết quả khác của giáo sư Vasile $$\frac{a(b+c)}{b^2+bc+c^2}+\frac{b(c+a)}{c^2+ca+a^2}+\frac{c(a+b)}{a^2+ab+b^2}\ge 2+4\cdot\left ( \frac{a-b}{a+b}\cdot\frac{b-c}{b+c}\cdot\frac{c-a}{c+a} \right )^2.$$


#363791 Cho $xyz=1$. Chứng minh rằng: $\sum\frac{x+3}{...

Gửi bởi Nguyenhuyen_AG trong 22-10-2012 - 09:50

Sử dụng bất đẳng thức AM-GM, ta có $$\frac{x}{(x+1)^2}+\frac{y}{(y+1)^2}+\frac{z}{(z+1)^2}\ge 3\sqrt[3]{\frac{xyz}{(x+1)^2(y+1)^2(z+1)^2}}=\frac{3}{\sqrt[3]{(x+1)^2(y+1)^2(z+1)^2}}$$ Mặt khác, theo bất đẳng thức AM-GM, thì $$(x+1)(y+1)(z+1)\ge 8.$$ Nên $$\frac{3}{\sqrt[3]{(x+1)^2(y+1)^2(z+1)^2}} \ge \frac{6}{(x+1)(y+1)(z+1)}.$$ Vậy để chứng minh bất đẳng thức trên, thì ta cần chứng minh được $$\frac{3}{(x+1)^2}+\frac{3}{(y+1)^2}+\frac{3}{(z+1)^2}+\frac{6}{(x+1)(y+1)(z+1)}\ge 3,$$ tương đương với $$\frac{1}{(x+1)^2}+\frac{1}{(y+1)^2}+\frac{1}{(z+1)^2}+\frac{2}{(x+1)(y+1)(z+1)}\ge 1.$$ Là một bất đẳng thức quen thuộc của Phạm Văn Thuận.


#360911 $\sum a^2+\sum ab^2+9 \geq 5\sum a$

Gửi bởi Nguyenhuyen_AG trong 11-10-2012 - 11:18

Đây là một bài toán của anh Quốc Anh, và nó nằm trong một chuyên đề mà mình đang viết. Xin giới thiệu với mọi người một phần của chuyên đề này cùng lời giải và những phân tích từ bài toán trên.

Chứng minh rằng nếu $a,b,c>0,$ thì $$a^2+b^2+c^2+ab^2+bc^2+ca^2+9 \ge 5(a+b+c).$$


Xem xét một chút về bất đẳng thức trên ta thấy, đây là một bài toán ba biến hoán vị. Và đại lượng hoán vị đó là $a{{b}^{2}}+b{{c}^{2}}+c{{a}^{2}}$ mà như chúng ta đã biết thì bất đẳng thức hoán vị thường khó xử lí hơn so với bất đẳng thức đối xứng. Nên ý tưởng của ta là sẽ tìm cách đưa bài toán hoán vị này về dạng đối xứng thông qua các đánh giá vào đại lượng $a{{b}^{2}}+b{{c}^{2}}+c{{a}^{2}}.$ Ta có các trường hợp sau đây.

Sử dụng bất đẳng thức Cauchy-Schwarz, ta có \[a{{b}^{2}}+b{{c}^{2}}+c{{a}^{2}}\ge \frac{{{\left( ab+bc+ca \right)}^{2}}}{b+c+a}.\quad (3.2)\] Còn theo bất đẳng thức AM-GM, thì \[( a+a{{b}^{2}} )+( b+b{{c}^{2}} )+( c+c{{a}^{2}})\ge 2\left( ab+bc+ca \right),\quad (3.3)\] \[a{{b}^{2}}+b{{c}^{2}}+c{{a}^{2}}\ge 3abc.\quad (3.4)\] Ta có ba cách đưa về đối xứng như trên và cả ba đều đảm bảo được dấu đẳng thức sẽ xảy ra tại $a=b=c=1.$

Với $(3.4),$ ta đưa bài toán về chứng minh \[{{a}^{2}}+{{b}^{2}}+{{c}^{2}}+3abc+9\ge 5\left( a+b+c \right),\quad (3.5)\]Tuy nhiên $(3.5)$ lại không đúng (có thể lấy ví dụ $a=0.\text{1},\text{ }b=\text{1},\text{ }c=\text{3})$ nên ta sẽ tìm cách chứng minh bài toán với hai đánh giá $(3.2)$ và $(3.3).$


Lời Giải 1. Sử dụng đánh giá \[a{{b}^{2}}+b{{c}^{2}}+c{{a}^{2}}\ge \frac{{{\left( ab+bc+ca \right)}^{2}}}{b+c+a}.\quad (3.6)\] Ta đưa bất đẳng thức về chứng minh \[{{a}^{2}}+{{b}^{2}}+{{c}^{2}}+\frac{{{\left( ab+bc+ca \right)}^{2}}}{a+b+c}+9\ge 5\left( a+b+c \right).\quad (3.7)\] Bất đẳng thức lúc này chứa 3 đại lượng là ${{a}^{2}}+{{b}^{2}}+{{c}^{2}},\text{}ab+bc+ca,\text{ }a+b+c$ mà chúng lại có liên quan với nhau thông qua hằng đẳng thức $${{\left( a+b+c \right)}^{2}}={{a}^{2}}+{{b}^{2}}+{{c}^{2}}+2\left( ab+bc+ca \right).\quad (3.8)$$ Vậy nếu ta xem mỗi một đại lượng trên là một biến thì thông qua đẳng thức (3.8) ta có thể biểu diễn được một biến thông qua hai biến còn lại, tức là sẽ đưa được bài toán ba biến trên thành bài toán hai biến và việc chứng minh một bất đẳng thức hai biến chắc chắn sẽ dễ chịu hơn là ba biến.
Với ý tưởng như vậy sử dụng hằng đẳng thức $${{\left( a+b+c \right)}^{2}}={{a}^{2}}+{{b}^{2}}+{{c}^{2}}+2\left( ab+bc+ca \right),\quad (3.9)$$ ta có thể viết bất đẳng thức chứng minh lại như sau \[{{\left( a+b+c \right)}^{2}}-2\left( ab+bc+ca \right)+\frac{{{\left( ab+bc+ca \right)}^{2}}}{a+b+c}+9\ge 5\left( a+b+c \right),\quad (3.10)\]
$$\left[ {{\left( a+b+c \right)}^{2}}-6\left( a+b+c \right)+9 \right]+\left[ \frac{{{\left( ab+bc+ca \right)}^{2}}}{a+b+c}-2\left( ab+bc+ca \right)+\left( a+b+c \right) \right]\ge 0,$$ hay là $${{\left( a+b+c-3 \right)}^{2}}+\frac{{{\left( ab+bc+ca-a-b-c \right)}^{2}}}{a+b+c}\ge 0.\quad (3.11)$$ Bất đẳng thức cuối cùng hiển nhiên đúng, nên bất đẳng thức trên đã được chứng minh xong.


Lời Giải 2. Ta có bất đẳng thức \[( a+a{{b}^{2}} )+( b+b{{c}^{2}} )+( c+c{{a}^{2}} )\ge 2\left( ab+bc+ca \right),\quad (3.12)\] \[a{{b}^{2}}+b{{c}^{2}}+c{{a}^{2}}\ge 2\left( ab+bc+ca \right)-\left( a+b+c \right).\quad (3.13)\] Sử dụng $(3.13)$ ta đưa bài toán về chứng minh \[{{a}^{2}}+{{b}^{2}}+{{c}^{2}}+2\left( ab+bc+ca \right)-\left( a+b+c \right)+9\ge 5\left( a+b+c \right),\quad (3.14)\] tương đương với \[{{\left( a+b+c \right)}^{2}}-6\left( a+b+c \right)+9\ge 0,\quad (3.15)\] \[{{\left( a+b+c-3 \right)}^{2}}\ge 0.\quad (3.16)\] Vậy bài toán đã được chứng minh xong.


Lời Giải 3. Cuối cùng chúng tôi xin được giới thiệu với các bạn một phương pháp rất hay thường dùng để chứng minh các bất đẳng thức dạng không thuần nhất này, đó là phương pháp đổi biến.

Vì đẳng thức của bài toán xảy ra tại $a=b=c=1$ nên ta sẽ đặt $$a=x+1,\text{ }b=y+1,\text{ }c=z+1. \quad (3.17)$$ Thay các giá trị này vào bất đẳng thức và thu gọn lại, ta được $$2({{x}^{2}}+{{y}^{2}}+{{z}^{2}})+2(xy+yz+zx)+x{{y}^{2}}+y{{z}^{2}}+z{{x}^{2}}\ge 0. \quad (3.18)$$ Bất đẳng thức trên có thể viết lại thành $${{\left( x+y+z \right)}^{2}}+{{y}^{2}}\left( x+1 \right)+{{z}^{2}}\left( y+1 \right)+{{x}^{2}}\left( z+1 \right)\ge 0,\quad (3.19)$$ $${{\left( x+y+z \right)}^{2}}+{{y}^{2}}a+{{z}^{2}}b+{{x}^{2}}c\ge 0.\quad (3.20)$$ Điều này là hiển nhiên đúng, vậy ta có điều phải chứng minh.


#360907 $$\sqrt{abc}(\sqrt{a}+\sqrt...

Gửi bởi Nguyenhuyen_AG trong 11-10-2012 - 11:03

Bài toán 1.
Ch0 các số thực dương $a,b,c$.Chứng minh rằng:
$$\sqrt{abc}(\sqrt{a}+\sqrt{b}+\sqrt{c})+(a+b+c)^2 \ge 4\sqrt{3abc(a+b+c)}$$


Trước hết, ta sẽ khử bớt đại lượng $3(a+b+c)$ trong căn thức bằng cách chuẩn hóa cho $a+b+c=3.$ Khi đó bất đẳng thức trở thành $$\sqrt{abc}(\sqrt{a}+\sqrt{b}+\sqrt{c})+9\ge 12\sqrt{abc}.$$ Ta sẽ gom biến về một bên, hằng số một bên bằng cách chia hai vế cho $\sqrt{abc}>0$ $$\sqrt{a}+\sqrt{b}+\sqrt{c}+\frac{9}{\sqrt{abc}}\ge 12.$$ Đến đây thì có lẽ lời giải đã tường minh, dễ thấy bất đẳng thức này có được bằng cách cộng hai bất đẳng thức sau đây lại với nhau $$\sqrt{a}+\sqrt{b}+\sqrt{c}+\frac{1}{\sqrt{abc}}\ge 4, \;\; \frac{8}{\sqrt{abc}} \ge 8.$$ Thế nhưng cả hai đều đúng theo bất đẳng thức AM-GM


#360899 Tìm min của $x^2+my^2+nz^2$

Gửi bởi Nguyenhuyen_AG trong 11-10-2012 - 10:31

Cho $x,y,z>0$ và $xy+yz+xz=1$.Tìm min của $P=x^2+ay^2+bz^2$

P/s: Bài này mình giải hoài ko đc :( .Lâu rồi ko onl VMF. Khởi động lại cái đã


Điều kiện của $a,b$ là phải dương thì phải. Bằng bất đẳng thức Cauchy-Schwarz, mình tìm được $P_{\min}=2t,$ với $t$ là nghiệm dương của phương trình $$f(x)=2x^3+(1+a+b)x^2-ab=0.$$ Thật vậy, giả sử rằng $P$ đạt giá trị nhỏ nhất là $2t >0,$ khi đó, ta được $$x^2+ay^2+bz^2 \ge 2t.$$ Tương đương với $$(t+1)x^2+(t+a)y^2+(t+b)z^2 \ge t(a+b+c)^2.$$ Sử dụng bất đẳng thức Cauchy-Schwarz, ta có $$(t+1)x^2+(t+a)y^2+(t+b)z^2\ge \frac{(x+y+z)^2}{\dfrac{1}{t+1}+\dfrac{1}{t+a}+\dfrac{1}{t+b}}.$$ Như vậy, ta cần chọn $t$ sao cho $$\dfrac{1}{t+1}+\dfrac{1}{t+a}+\dfrac{1}{t+b}=\frac{1}{t}. \quad (1)$$ Bằng cách quy đồng và thu gọn, ta thấy $(1)$ tương đương với $$2t^3+(1+a+b)t^2-ab=0.$$

Nhận Xét. Với $a=\frac{2}{5},\;b=\frac{1}{5}$ ta được một kết quả rất đẹp sau đây $$5a^2+2b^2+c^2\ge 2(ab+bc+ca).$$ Với đẳng thức xảy ra khi $(a,b,c)$ tỉ lệ với $(1,2,3).$


#360889 CMR: $xyz(x-1)(y-1)(z-1) ≤8$

Gửi bởi Nguyenhuyen_AG trong 11-10-2012 - 10:15

Với những bất đẳng thức có dạng Shur như $$P(a,b,c)\cdot abc \ge F(a,b,c) \cdot (a+b-c)(b+c-a)(c+a-b),$$ thì S-S là một kỹ thuật áp dụng tương đối tốt cho các dạng toán này. http://mathifc.wordp...thuc-dang-shur/


#359525 $\frac{a\left ( b+c \right )}{b^{2}+bc+c^{2}}+\frac{...

Gửi bởi Nguyenhuyen_AG trong 06-10-2012 - 20:12

Cho a,b,c là các số không âm.
Chứng minh rằng:$\frac{a\left ( b+c \right )}{b^{2}+bc+c^{2}}+\frac{b\left ( a+c \right )}{c^{2}+ac+a^{2}}+\frac{c\left ( a+b \right )}{a^{2}+ab+b^{2}}\geq 2$

Chúng ta còn có bất đẳng thức mạnh hơ sau đây $$\frac{a\left ( b+c \right )}{b^{2}+bc+c^{2}}+\frac{b\left ( a+c \right )}{c^{2}+ac+a^{2}}+\frac{c\left ( a+b \right )}{a^{2}+ab+b^{2}}\geq 2+\frac{(a-b)^2(b-c)^2(c-a)^2}{(a^2b+b^2c+c^2a)(ab^2+bc^2+ca^2).}$$


#349743 $$\dfrac{(-a+b+c)^2}{b^2+c^2}+\dfrac...

Gửi bởi Nguyenhuyen_AG trong 25-08-2012 - 23:58

Bài này trên mathlinks, bạn kunny người Nhật Bản có một lời giải khá hay trong blog cá nhân như sau http://www.artofprob....com/blog/43938


#346438 Tìm GTLN của : $3b+8c+abc$

Gửi bởi Nguyenhuyen_AG trong 13-08-2012 - 13:09

Sử dụng bât đẳng thức AM-GM, ta có $$ \dfrac{b^2+1}{2}\ge b,\;\dfrac{c^2+1}{2}\ge c.$$ Sử dụng đánh giá này, ta đưa bài toán về chứng minh bất đẳng thức mạnh hơn sau đây $$ 3\cdot\dfrac{b^2+1}{2}+8\cdot\dfrac{c^2+1}{2}+abc\le 12,$$ hay là $$ 3b^2+8c^2+2abc\le 13.$$ Thuần nhất hai vế của bất đẳng thức, ta được $$ 3b^2+8c^2+2abc\sqrt{\dfrac{14}{a^2+4b^2+9c^2}}\le 13\cdot\dfrac{a^2+4b^2+9c^2}{14},$$ thu gọn thành $$ 13a^2+10b^2+5c^2\ge2abc\sqrt{\dfrac{14}{a^2+4b^2+9c^2}},$$ hoặc $$ (a^2+4b^2+9c^2)(13a^2+10b^2+5c^2)^2\ge28^2\cdot14\cdot(abc)^2,$$ $$ \displaystyle\left(\frac{a^2}{14}+\frac{4b^2}{14}+\frac{9c^2}{14}\right)\left(\frac{13a^2}{28}+\frac{10b^2}{28}+\frac{5c^2}{28}\right)^2\ge a^2b^2c^2.$$ Thế nhưng bất đẳng thức này đúng vì theo bất đẳng thức AM-GM suy rộng, ta có $$ \displaystyle\frac{a^2}{14}+\frac{4b^2}{14}+\frac{9c^2}{14}\ge (a^2)^{\frac{1}{14}}\cdot(b^2)^\frac{4}{14}\cdot(c^2)^\frac{9}{14}.$$ $$ \begin{aligned}\left(\frac{13a^2}{28}+\frac{10b^2}{28}+\frac{5c^2}{28}\right)^2&\ge (a^2)^{2\cdot\frac{13}{28}}\cdot(b^2)^{2\cdot\frac{10}{28}}\cdot(c^2)^{2\cdot\frac{5}{28}}\\&= (a^2)^{\frac{13}{14}}\cdot(b^2)^{\frac{10}{14}}\cdot(c^2)^{\frac{5}{14}}.\end{aligned}$$ Nhân hai bất đẳng thức trên lại với nhau, ta được $$ \begin{aligned}\left(\frac{a^2}{14}+\frac{4b^2}{14}+\frac{9c^2}{14}\right)\left(\frac{13a^2}{28}+\frac{10b^2}{28}+\frac{5c^2}{28}\right)^2&\ge(a^2)^{\frac{1}{14}}\cdot(b^2)^\frac{4}{14}\cdot(c^2)^\frac{9}{14}\cdot(a^2)^{\frac{13}{14}}\cdot(b^2)^{\frac{10}{14}}\cdot(c^2)^{\frac{5}{14}}\\&= (a^2)^{\frac{1}{14}+\frac{13}{14}}\cdot(b^2)^{\frac{4}{14}+\frac{10}{14}}\cdot(c^2)^{\frac{9}{14}+\frac{5}{14}}\\&=a^2b^2c^2.\end{aligned}$$ Bài toán được chứng minh xong, đẳng thức xảy ra khi $ a=b=c=1.\Box$
P/s. Đây là đề thì 30/4 năm 2011 của anh Cẩn ra cho các bạn lớp 10.